Which statement accurately describes the inequalities below?

(i) [tex]-\frac{4}{3} \ \textgreater \ -1.3[/tex]
(ii) [tex]\frac{1}{2} \ \textless \ 0.5[/tex]

A. (i) is true and (ii) is true.
B. (i) is false and (ii) is false.
C. (i) is false and (ii) is true.
D. (i) is true and (ii) is false.



Answer :

To determine which statement accurately describes the given inequalities, let’s analyze each one step-by-step.

Inequality (i): [tex]\(-\frac{4}{3} > -1.3\)[/tex]

- To compare [tex]\(-\frac{4}{3}\)[/tex] and [tex]\(-1.3\)[/tex], we convert [tex]\(-\frac{4}{3}\)[/tex] into decimal form:
[tex]\[ -\frac{4}{3} \approx -1.3333... \][/tex]
- Comparing [tex]\(-1.3333...\)[/tex] and [tex]\(-1.3\)[/tex], we see that [tex]\(-1.3333...\)[/tex] is less than [tex]\(-1.3\)[/tex].
- Therefore, the inequality [tex]\(-\frac{4}{3} > -1.3\)[/tex] is false.

Inequality (ii): [tex]\(\frac{1}{2} < 0.5\)[/tex]

- Convert [tex]\(\frac{1}{2}\)[/tex] into decimal form:
[tex]\[ \frac{1}{2} = 0.5 \][/tex]
- Comparing [tex]\(0.5\)[/tex] with [tex]\(0.5\)[/tex], they are equal.
- Therefore, the inequality [tex]\(\frac{1}{2} < 0.5\)[/tex] is false.

Given that both inequalities are false, we check which statement describes this situation accurately:

- (i) is false and (ii) is false.

So, the correct statement is:
(i) is false and (ii) is false.